LSAT and Law School Admissions Forum

Get expert LSAT preparation and law school admissions advice from PowerScore Test Preparation.

 Administrator
PowerScore Staff
  • PowerScore Staff
  • Posts: 8917
  • Joined: Feb 02, 2011
|
#82062
Complete Question Explanation

The correct answer choice is (C).

Answer choice (A):

Answer choice (B):

Answer choice (C): This is the correct answer choice.

Answer choice (D):

Answer choice (E):


This explanation is still in progress. Please post any questions below!
 jmramon
  • Posts: 47
  • Joined: Jul 21, 2017
|
#37840
Hi, Powerscore!

I am unsure why answer choice "C" is correct over "E".

My interpretation of the mode of reasoning of passage A: Recent neurological studies discovered that one can be rational yet lack free will devoid of emotional causation. Due to these findings that undermine the law's current understanding of rational human behavior, the criminal justice system should replace its retributive justice measures with ones instead focused on deterring future crime. In brief: We should abandon current methods that fail to achieve their stated goal for new methods.

Why "C" is likely correct/summary of this answer: The currently misguided system (in this case, the existing program for teaching math in elementary schools), should be abandoned for another system. This closely mirrors passage A's argument in conceptual terms and this is likely why "C" is correct.

Summary of "E"/why this answer is incorrect: Just because one has free will doesn't mean they can exercise that free will in all situations; addiction is a case in point. This answer does not mirror the prephase of passage A in any way, but does conceptually mirror premises in passage A. This answer is likely wrong because the overall argument of passage A (abandoning current methods that fail to achieve their stated goal for new methods) isn't conceptually present.

Is my understanding of why "C" is correct and "E" is incorrect accurate?

Thank you, Powerscore!:)
User avatar
 Jonathan Evans
PowerScore Staff
  • PowerScore Staff
  • Posts: 726
  • Joined: Jun 09, 2016
|
#37853
Hi, jmramon,

Thanks for the question and for the detailed explanations of your analysis. Your explanation of (C) is more or less spot on: it discusses an analogous mistaken rationale for a program that should therefore be replaced with a superior program.

You are more or less right about (E); in addition, however, note that (E) lacks the degree of specificity that (C) has. While (E) might sync up with some of the discussions of concepts in Passage A (as it also does with certain concepts discussed in Passage B), there is not direct correlation between the concepts in question between answer choice (E) and the argument advanced in Passage A. The question stem asks about "the" argument in Passage A; therefore, the credited response will deal with the specific argument about the criminal justice system and not just conceptual matches from elsewhere in the passage.

Good question! Please follow up with more. I hope this helps!
 jmramon
  • Posts: 47
  • Joined: Jul 21, 2017
|
#37952
This helps immensely! Thank you very much, Jonathan!
 Khodi7531
  • Posts: 116
  • Joined: Mar 14, 2018
|
#45782
I was between B and C and chose B. I thought B had a crucial component missing in C that was evidence. A abandons an idea by new evidence. However, I didn't catch at the end where B says,"should not be able to predict" because that's not the same as you, you should abandon.

C says the right things, but not "enough". That's a hard thing for me to get used to...answers that don't state enough. But should get to it by POE. Is that a good enough reason to chose B over C?
 Adam Tyson
PowerScore Staff
  • PowerScore Staff
  • Posts: 5153
  • Joined: Apr 14, 2011
|
#49737
It is, khodi! Once you have eliminated all the answers that cannot be right, whatever is left must be. However, your concerns about answer C appear to be misplaced to me. The argument in passage A has a lot of detail, obviously, but it still boils down to "the current dominant model of punishment as retribution should be abandoned because it is based on a misunderstanding of what drives people to commit crimes, and a new model based on deterrence should be adopted." Answer C captures that same "based on misunderstanding, so replace" structure. We don't need all the details to match, just the underlying abstract structure of the arguments.
 snowy
  • Posts: 73
  • Joined: Mar 23, 2019
|
#64485
Could someone elaborate on why C is correct over E, please? The discussion by jmramon and Jonathan above helped me start to understand, but not yet completely. Thank you!!
 Zach Foreman
PowerScore Staff
  • PowerScore Staff
  • Posts: 91
  • Joined: Apr 11, 2019
|
#64499
Snowy,
Again, it is all about the prephrase. How did you condense the argument of passage a? How do you make it more abstract?
Neuroscience tells us that while we may be rational, our actions are determined and are not freely chosen. So law needs to move away from punishing immoral acts but rather focus on reducing future harms."
Now, more abstract:
New information should cause us to change the way something is done.
C fits that very well, but E is missing the point entirely, not having anything to say about changing the way something is currently done nor about new information being gained.

Finally, I would always be cautions when a parallel answer choice parallels the content of the passage. This is usually the wrong answer since we are looking for parallels in argument shape not content.
 acp25
  • Posts: 13
  • Joined: Sep 21, 2017
|
#68479
Hi, All,

Can someone please explain to me why answer choice A is incorrect?

I did not prephrase passage A accurately: we should abandon an idea if it is no longer relevant. I missed the part where something must be replaced. By knowing what I missed, I now understand why C is correct.

Is A incorrect because it only implies abandoning useless features, and says nothing about replacing?

I hope my explanation made sense,
A.
 James Finch
PowerScore Staff
  • PowerScore Staff
  • Posts: 943
  • Joined: Sep 06, 2017
|
#70974
Hi ACP,

In a parallel question, we have to closely match/double both the premises and conclusion of the original argument. (A) doesn't get to the heart of the argument in passage A, which is that the current system is based on mistaken assumptions that science has shown to be false, thus implying that the current system should be replaced with one based upon more up-to-date information. (A) instead deals with extraneous features, which should be cut down. But the passage isn't talking about too many features getting in the way of use, and thus getting rid of the features increasing productivity by making something easier to use, but instead about new information/understanding about human thought rendering a system based on mistaken assumptions about human thought obsolete.

Hope this clears things up!

Get the most out of your LSAT Prep Plus subscription.

Analyze and track your performance with our Testing and Analytics Package.